10
$\begingroup$

I'm hoping to find a reasonable value to assign to the divergent series $\sum_{n=0}^\infty (-1)^n n^n$ and $\sum_{n=0}^\infty (-1)^n (xn)^n$. For the first one, I have obtained something around 0.71, but I'm very unsure if this is correct. For the second series, I get a graph that looks about like this: enter image description here.

What sort of methods are powerful enough to sum this series? I think Borel summation is too weak, since this grows faster than any $\left(\alpha n\right)!$.

Any help would be appreciated!

EDIT: I obtained my values from a series of questionably valid approximations, but here is another method that seems to give a similar value.

The idea is that illegally swapping summation signs, and then continuing the inner summation past its regular convergent range allows one to assign a value to a divergent series.

Starting with $$\sum_{n=0}^\infty (-1)^n n^n$$ We want to get two summations in order to be able to swap, so we expand into $$\sum_{n=0}^\infty (-1)^n e^{n\ln(n)} = \sum_{n=0}^\infty (-1)^n \sum_{k=0}^\infty \frac{\left(n \ln(n)\right)^k}{k!}$$ Simplifying a bit more, we get $$\sum_{n=0}^\infty (-1)^n \sum_{k=0}^\infty \frac{n^k \ln(n)^k}{k!} =\sum_{n=0}^\infty (-1)^n \sum_{k=0}^\infty \frac{e^{\ln(n)k} \ln(n)^k}{k!}$$ Swapping the summations, we get $$\sum_{k=0}^\infty \frac{1}{k!} \sum_{n=0}^\infty (-1)^n e^{\ln(n)k} \ln(n)^k = \sum_{k=0}^\infty \frac{1}{k!} \sum_{n=0}^\infty \frac{d^k}{dk^k} (-1)^n n^k $$ (Note: I'm using $\frac{d^k}{dk^k}$ to represent taking the derivative with respect to k, k times). Now, we can continue the inner summation by doing $$\sum_{k=0}^\infty \frac{1}{k!} \frac{d^k}{dk^k}\sum_{n=0}^\infty (-1)^n n^k = \sum_{k=0}^\infty \frac{1}{k!} \frac{d^k}{dk^k}(1-\eta(-k))$$ To finally get $$1/2 - (\eta'(-1) - \frac{\eta''(-2)}{2!} + \frac{\eta'''(-3)}{3!} - \dots)$$ The sum of the first few terms seems to be around .71, which agrees fairly closely with the other method.

EDIT2: It looks like most methods converge onto about the same shape eventually. enter image description here The green line is the asymptotic expansion, meaning I only took the first 5 terms in the power series, and left the rest out. Usually, this asymptotic expansion converges to the right function within a very small radius. The black function is using something like FusRoDah's method. The orange graph is using the non-rigourous series of approximations.

This next pictures shows the functions over larger intervals--the purple function is using that eta method I outlined above.

enter image description here

To elaborate more about how I used FusRoDah's method, I started with $BA(t)=\sum_{k=0}^\infty \frac {(-kt)^k} {k!} = 1+\sum_{k=1}^\infty \frac {(-kt)^k} {k!}$. Then, I added in the approximation to get $$\operatorname{BA}(t) = 1+\sum_{k=1}^\infty \frac {(-kt)^k} {k!} + \frac {(-et)^k} {\sqrt{2\pi k}}-\frac {(-et)^k} {\sqrt{2\pi k}} = 1+\sum_{k=1}^\infty\frac {(-et)^k} {\sqrt{2\pi k}}+ \sum_{k=1}^\infty \frac {(-kt)^k} {k!} -\frac {(-et)^k} {\sqrt{2\pi k}}.$$ The first part of the sum can be written as an integral of the polylogirthmn with some other terms, but we are still left with the $\int_{0}^{\infty}e^{-t}\sum_{k=1}^{\infty}\left(\frac{\left(-ktz\right)^{k}}{k!}-\frac{\left(-etz\right)^{k}}{\sqrt{2\pi k}}\right)dt$ term. Since this doesn't converge on its own, we can approximate it with $$\int_{0}^{A}e^{-t}\sum_{k=1}^{B}\left(\frac{\left(-ktz\right)^{k}}{k!}-\frac{\left(-etz\right)^{k}}{\sqrt{2\pi k}}\right)dt.$$ Increasing $A$ and $B$ increases the accuracy, but decreases the range of convergence. I used $A = 2.5$, $B = 30$ for the first graph.

What remains to be done then is either find a more accurate approxation, such that $\sum_k \frac{(-kt)^k}{k!} - f_t(k)$ converges for all values of t, or to find a way to continue the difference between the approximation and the original series.

$\endgroup$
6
  • 2
    $\begingroup$ Could you elaborate on how you obtained the 0.71 value? $\endgroup$
    – Max Muller
    Sep 6, 2021 at 20:29
  • $\begingroup$ @MaxMuller I obtained .71 from a series of very unrigorous approximations, that is why I'm hoping someone can provide a more reasonable solution so I can figure out which parts of my approximations are reasonble and which parts are not. However, I'm thinking of another method to solve this that I will edit in, but its similarly unrigorous $\endgroup$ Sep 6, 2021 at 20:35
  • 2
    $\begingroup$ I think that it follows from Stirling's approximation that $n!<n^n e^n$, so Borel summation should be fine. However, expecting some kind of "nice" formula for the sum is probably too optimistic. $\endgroup$
    – FusRoDah
    Sep 6, 2021 at 21:05
  • $\begingroup$ For some years I found with my methods $w \approx 1- 0.29632 = 0.70368$ I used as well the double-summation-formula which occurs by expanding the implicite exponential-series, and also applied a (Carleman-)matrix approach. I did never improve that purely explorative draft computation and discussion from 2007 (go.helms-net.de/math/tetdocs/Tetra_Etaseries.pdf) , maybe there's something interesting in it for you. If there is something in it, I'd propose to use email-conversation instead of littering your question here. helms (at) uni-kassel.de $\endgroup$ Sep 9, 2021 at 9:56
  • 1
    $\begingroup$ The sum-formula beginning with $\sum \frac1{k!} \sum \cdots$ can be evaluated using Pari/GP's sumalt()-function for the inner alternating sums followed by a simple Euler-summation of that intermediate results with small order. Using $64$ terms for the outer sum I have this Pari/GP-formulation: tmp=Mat(vectorv(64,k,1/k!*sumalt(n=1,(-1)^n*n^k*log(n)^k))) and for display ESum(1.3,64)*tmp + 0.5*mV(1,64) giving the $60...64$'th approximations $$ 0.70416996043747446070 \\ 0.70416996043747446035 \\ 0.70416996043747446017 \\ 0.70416996043747446011 \\$$ $\endgroup$ Sep 13, 2021 at 11:58

2 Answers 2

17
$\begingroup$

The sum is equal to $$ \int\limits_{0}^{\infty}\frac{\exp(-x)}{1+W_0(x)}\,\mathrm{d}x = 0.7041699604... $$ where $W_0(x)$ is the Lambert-$W$ function.

Reference

Stephen Finch. "Errata and Addenda to Mathematical Constants [Math.HO]", §6.11. Iterated Exponential Constants, pg. 66.

Two interesting divergent sums from this reference are as well $$\sum_{n=1}^\infty (-1)^{n-1} (2n)^{2n-1} = \int\limits_{0}^{\infty}e^{-x}\log(x/|W_{0}(i x)|)\mathrm{d}x = 0.3233674316...$$ which seems to be also the cosine and sine integrals of the derivatives $W'_{0}(x) = \frac {W_{0}(x)}{x(1 + W_{0}(x))}$ and $-W''_{0}(x)$ respectively $$\sum_{n=1}^\infty (-1)^{n-1} (2n)^{2n-1} = \int\limits_{0}^{\infty}\mathrm{cos}(x) W'_{0}(x)\mathrm{d}x = -\int\limits_{0}^{\infty}\mathrm{sin}(x) W''_{0}(x)\mathrm{d}x$$ as it can be seen in the following Pari GP v2.14.0 lines

Alternating sum of even powered to odd numbers

although, as it is pointed out, a rigorous proof is not yet known.

And this variation, $$\sum_{n=1}^\infty (-1)^{n-1} (2n-1)^{2n} = \frac{1}{2i}\int\limits_{0}^{\infty}e^{-x}(g(ix)-g(-ix))\mathrm{d}x = 0.0111203007...$$ where $g(x) = W_{0}(x)/(1 + W_{0}(x))^3$

$\endgroup$
5
  • 3
    $\begingroup$ Wow! This is truly incredible-- thank you for this answer! $\endgroup$ Sep 7, 2021 at 3:17
  • 2
    $\begingroup$ I should add to this wonderful answer, that $$\int_0^\infty \frac{e^{-x}}{1+W_0(x)}dx = \int_1^\infty \frac{1}{x^x}$$ Which makes this answer answer very similar to form to the sophmore's dream $\endgroup$ Sep 7, 2021 at 16:16
  • $\begingroup$ I thought about this method, but did not know how to extend $\sum _{k=0}^{\infty } \frac{(-1)^k k^k t^k}{k!}$ analytically. How to prove that this sum is $1/(1+W_0(t))$? $\endgroup$ Sep 8, 2021 at 14:01
  • 2
    $\begingroup$ Iosif, the derivative relationship 1 - t*dW0(t)/dt =1/(1+W0(t)) makes the trick. This sum is an excellent probe test for summation methods acting on alternating divergent series. $\endgroup$ Sep 9, 2021 at 0:01
  • $\begingroup$ @JorgeZuniga : Thank you for your response. $\endgroup$ Sep 9, 2021 at 23:39
2
$\begingroup$

The sum we consider is $\sum_{n=1}^\infty (-nz)^n$. (The $n=0$ term causes problems later on.) The Borel transform of this sum is $BA(t)=\sum_k \frac {(-kt)^k} {k!}$. I don't see how we could make progress with this sum, but using Stirling's formula, it should be almost equal to $\sum_k \frac {(-et)^k} {\sqrt{2\pi k}}$. This series only converges for $|t|<1/e$, but can be analytically continued to the so-called polylogarithm $(2\pi)^{-1/2} Li_{1/2}(-et)$. This can the be integrated in the usual way, in which one proceeds with Borel summation. In the case $z=1$, a quick WolframAlpha computation gives $-0.33645$, which should be close to the "value" of your sum.

$\endgroup$
1
  • $\begingroup$ I think your value that you assign to the sum agrees with mine, I'll post more details after I finish working it out $\endgroup$ Sep 6, 2021 at 22:28

Your Answer

By clicking “Post Your Answer”, you agree to our terms of service and acknowledge you have read our privacy policy.

Not the answer you're looking for? Browse other questions tagged or ask your own question.